Paper 3, Section II, 29K

Stochastic Financial Models | Part II, 2020

(a) Let (Bt)t0\left(B_{t}\right)_{t \geqslant 0} be a real-valued random process.

(i) What does it mean to say that (Bt)t0\left(B_{t}\right)_{t \geqslant 0} is a Brownian motion?

(ii) State the reflection principle for Brownian motion.

(b) Suppose that (Bt)t0\left(B_{t}\right)_{t \geqslant 0} is a Brownian motion and set Mt=supstBsM_{t}=\sup _{s \leqslant t} B_{s} and Zt=MtBtZ_{t}=M_{t}-B_{t}.

(i) Find the joint distribution function of BtB_{t} and MtM_{t}.

(ii) Show that (Mt,Zt)\left(M_{t}, Z_{t}\right) has a joint density function on [0,)2[0, \infty)^{2} given by

P(Mtdy and Ztdz)=22πt(y+z)te(y+z)2/(2t)dydz\mathbb{P}\left(M_{t} \in d y \text { and } Z_{t} \in d z\right)=\frac{2}{\sqrt{2 \pi t}} \frac{(y+z)}{t} e^{-(y+z)^{2} /(2 t)} d y d z

(iii) You are given that two of the three processes (Bt)t0,(Mt)t0\left(\left|B_{t}\right|\right)_{t \geqslant 0},\left(M_{t}\right)_{t \geqslant 0} and (Zt)t0\left(Z_{t}\right)_{t \geqslant 0} have the same distribution. Identify which two, justifying your answer.

Typos? Please submit corrections to this page on GitHub.